Simplifying a sum involving lnx

  • Thread starter Thread starter dustbin
  • Start date Start date
  • Tags Tags
    Simplifying Sum
Click For Summary
The discussion revolves around proving that the limit as n approaches infinity of the expression involving the sum of ln(k) minus ln(n) is equivalent to another limit involving ln(k/n). The initial attempt involved recognizing that the sum of ln(k) from k=1 to n equals ln(n!). A key insight was rewriting the expression as (1/n)ln(n!/n^n), which simplifies the problem. Participants clarified the correct logarithmic properties and the importance of the order of summation in understanding the limits. Ultimately, the solution was reached by reversing the order of the logarithmic terms in the sum.
dustbin
Messages
239
Reaction score
6

Homework Statement



I need to show that the lim n->inf ( [(1/n) * sum from k=1 to n of ln(k) ] - ln(n) )
is equal to the lim n->inf ( (1/n) sum from k=1 to n of ln(k/n) )



The Attempt at a Solution


I showed that the sum of ln(n) from k=1 to n is ln(n!) using ln(a) + ln(b) = ln(a+b). I am not sure how to get from (1/n)ln(n!) - ln(n) to (1/n)sum-ln(k/n).


I apologize for the non-tex stuff... but I am not sure how to do some of the symbols for this in tex yet. Thank you for your help.
 
Physics news on Phys.org
ln(a) + ln(b) ≠ ln(a+b), but ln(a) + ln(b) = ln(ab).
Try working with that.
 
Whoops. Actually that was a typo in my first post. I know that ln(a)+ln(b) ≠ ln(a+b):redface:
 
Did you try calculating both the limits seperately?
 
I have found a solution to this. Continuing from where you left, i can write it as:
\frac{\ln (n!)-n\ln (n)}{n}
which is equal to
\frac{\ln (\frac{n!}{n^n})}{n}

We can write the last term as
\frac{1}{n}(\ln \frac{1}{n}+\ln \frac{2}{n}+\ln \frac{3}{n}...
 
Oooh. I got up to (1/n)ln(n!/n^n) but did not see that it gives what you have for the last step. After thinking about it I can see why this is. Also, I did try to calculate both limits separately... but I could not find a way to the solution by doing so. I appreciate your input.

I was looking at ln(n!) as [ ln(n) + ln(n-1) + ln(n-2) + ... + ln(2) + ln(1) ] instead of in the opposite order, or [ ln(1) + ln(2) + ln(3) + ... ln(n-1) + ln(n) ]. After I reversed the order, I saw how you obtained the solution :-P

Thank you for your input and help.
 
Question: A clock's minute hand has length 4 and its hour hand has length 3. What is the distance between the tips at the moment when it is increasing most rapidly?(Putnam Exam Question) Answer: Making assumption that both the hands moves at constant angular velocities, the answer is ## \sqrt{7} .## But don't you think this assumption is somewhat doubtful and wrong?

Similar threads

  • · Replies 5 ·
Replies
5
Views
1K
Replies
12
Views
3K
  • · Replies 4 ·
Replies
4
Views
1K
  • · Replies 2 ·
Replies
2
Views
1K
Replies
7
Views
1K
  • · Replies 22 ·
Replies
22
Views
4K
  • · Replies 4 ·
Replies
4
Views
1K
Replies
11
Views
2K
  • · Replies 1 ·
Replies
1
Views
2K
  • · Replies 2 ·
Replies
2
Views
2K